Jump to content

Trang Luong's Content

There have been 1000 items by Trang Luong (Search limited from 06-06-2020)



Sort by                Order  

#493178 Xử lí vi phạm của các mem mới gia nhập

Posted by Trang Luong on 15-04-2014 - 21:39 in Xử lí vi phạm - Tranh chấp - Khiếu nại

Thưa BQT, em thấy những thành viên có danh hiệu là Lính mới . Đăng nhiều bài lỗi. Em chỉ sửa chữa, nhắc nhở nhẹ .Nhưng em thấy mấy mod ghi điểm thì em thấy nặng quá có thể làm những thành viên mới k có sở thích với diễn đàn mình. 

Vậy em nên nhắn tin nhắc nhở nhẹ hay là phải ghi điểm nhắc nhở ạ. Em cảm ơn BQT




#493589 Xử lí vi phạm của các mem mới gia nhập

Posted by Trang Luong on 17-04-2014 - 21:08 in Xử lí vi phạm - Tranh chấp - Khiếu nại

Đề nghị buiminhhieu nếu đã khóa topic thì khóa những bài nên khóa thôi, và cũng ẩn đi cho box đỡ chật. Nếu k khóa thì sửa chữa chứ. Toàn để thế thì sao đươc




#493599 Xử lí vi phạm của các mem mới gia nhập

Posted by Trang Luong on 17-04-2014 - 21:23 in Xử lí vi phạm - Tranh chấp - Khiếu nại

Mình thấy các mem mới phải tự sửa tiêu đề lỗi mà mình gây ra chứ 

Đây

BQT lưu ý các ĐHV. Khi các mem nhờ các ĐHV mở hộ topic bị khóa, ĐHV cần sử dụng chức năng nhắc nhở để nhắc nhở mem đó, sau đó, ĐHV cần yêu cầu mem đó tự sửa tiêu đề.

 

Topic sai tiêu đề bị khóa quá 3 ngày mà không có ai xin mở thì có thể xóa

nhưng toàn mấy mem mắc lỗi lần đầu thì sửa chữa thôi. Khóa làm gì, gởi tin nhắc nhở 




#453021 Xử lí của các mod THCS

Posted by Trang Luong on 25-09-2013 - 21:18 in Xử lí vi phạm - Tranh chấp - Khiếu nại

Dạo này em thấy các mod THCS làm việc k hiệu quả cho lắm. Nhiều bài k đủ chất lượng chỉ đê câu số lượng bài, hay nhiều bạn nói chuyện trên các topic, hay cả đăng những bài báo toán tuổi trẻ và tuổi thơ nữa http://diendantoanho...-r/#entry452949

dù trong giờ đó có nhiu mod vẫn đang hoạt động và xử lý thực sự rất chậm. Em trân thành góp ý mong các mod đừng có để bụng. 




#579849 Xét hàm số $f(x)={{x}^{3}}+a{{x...

Posted by Trang Luong on 08-08-2015 - 21:49 in Phương trình - Hệ phương trình - Bất phương trình

Gọi  $a,b,c$ là các số nguyên có giá trị tuyệt đối không vượt quá $10$. Xét hàm số $f(x)={{x}^{3}}+a{{x}^{2}}+bx+c$ thỏa mãn $|f(2+\sqrt{3})|<0,0001$.Hỏi $2+\sqrt{3}$có thể là nghiệm của $f$ được không ?

Cho $c=0, a=-4,b=1$, vậy có tồn tại




#548664 Xác định $k$ để dãy là số chính phương

Posted by Trang Luong on 22-03-2015 - 10:01 in Dãy số - Giới hạn

Cho dãy $(x_{n})$ xác định với $x_{1}=x_{2}=1$ và $x_{n+2}=(4k-5)x_{n+1}-x_{n}+4-2k$.

Xác định $k\in Z$ để dãy là số chính phương.




#412845 x-4\sqrt{1-x}+m= 0

Posted by Trang Luong on 15-04-2013 - 20:22 in Đại số

Cho phương trình $x-4\sqrt{1-x}+m= 0$ (1) (x là ẩn số)

 

Tìm giá trị của m để (1) có nghiệm

Ta có:

$x-4\sqrt{1-x}+m=0$                         ĐK $x\leq 1$

$\Leftrightarrow 1-x+4\sqrt{1-x}+4-m-5=0$

$\Leftrightarrow (\sqrt{1-x}+2)^{2}=m+5$

mà $\sqrt{1-x}+2\geq 2$ và $\sqrt{1-x}+2\geq 2\Rightarrow m+5\geq 4\Rightarrow m\geq -1$

do đó $m\geq -1$

Nếu $m< -1\Rightarrow \sqrt{1-x}+2<2$ (vô lý)

Vậy $m\geq -1$




#413088 x-4\sqrt{1-x}+m= 0

Posted by Trang Luong on 16-04-2013 - 21:44 in Đại số

nếu m=0 thì phương trình có nghiệm $\sqrt{1-x}= \sqrt{5}-2\Rightarrow x=1-\left ( \sqrt{5}-2 \right )^{2}$

nếu $m\neq 0$ thì ....

bạn đã hiểu nhầm đề bài. Bài này hỏi khoảng giá trị của m để tìm đc x. Chứ ko phải tìm giá trị của m. Vì x luôn thay đổi và mỗi giá m sẽ có 1 giá trị x tương ứng




#451548 x+y+z$\geq$$\frac{3}{2}$ tì...

Posted by Trang Luong on 18-09-2013 - 20:53 in Bất đẳng thức và cực trị

cho x;y;z >0 thoả mãn :

x+y+z$\geq$$\frac{3}{2}$

tìm giá trị nhỏ nhất của tổng:

x+y+z+$\frac{1}{x}+\frac{1}{y}+\frac{1}{z}$

thank nhiều :icon6:  :icon6:  :like  :like  :like  :like  :like  :like

Áp dụng BĐT AM-GM

Ta có : $x+y+z+\frac{1}{x}+\frac{1}{y}+\frac{1}{z}\geq (x+y+z)+\frac{9}{x+y+z}=(x+y+z)+\frac{9}{4(x+y+z)}+\frac{27}{4(x+y+z)}\geq 3+\frac{27}{4.\frac{3}{2}}=\frac{15}{2}$




#440598 x(x+2) + 4y(y-1) = 3

Posted by Trang Luong on 05-08-2013 - 14:34 in Đại số

Tìm nghiệm nguyên của pt:

x(x+2) + 4y(y-1) = 3

$\Leftrightarrow x^{2}+2x+1+4y^{2}-4y+1=5$$\Leftrightarrow (x+1)^{2}+(2y-1)^{2}=5$

Vì $(x+1)^{2}$ và $(2y-1)^{2}$ không âm nên xét 2 trường hợp $(x+1)^{2}=1;(2y-1)^{2}=4$ hoặc $(x+1)^{2}=4;(2y-1)^{2}=1$




#495992 Với mọi số nguyên dương $m,n$ thì $\frac{(2m)!(2...

Posted by Trang Luong on 29-04-2014 - 22:07 in Số học

Với mọi số nguyên dương $m,n$ thì $\frac{(2m)!(2n)!}{(m+n)!m!n!}\epsilon \mathbb{Z}$




#493938 Với a,b,c là số dương. Chứng minh $\frac{ab}{c}...

Posted by Trang Luong on 19-04-2014 - 19:49 in Bất đẳng thức và cực trị

Với a,b,c là số dương. Chứng minh $\frac{ab}{c}+\frac{bc}{a}+\frac{ac}{b}\geq a+b+c$

Ta cm BĐT : $a^2b^2+b^2c^2+c^2a^2\geq abc(a+b+c)$ luôn đúng

Theo BĐT AM-GM

$\Rightarrow a+b+c\leq \frac{a^2b^2+c^2b^2+c^2a^2}{abc}=\frac{ab}{c}+\frac{cb}{a}+\frac{ac}{b}$




#452596 Về dùng đường dẫn link

Posted by Trang Luong on 23-09-2013 - 19:04 in Hướng dẫn - Trợ giúp - Giải đáp thắc mắc khi sử dụng Diễn đàn

Đây




#516511 USA USAMO 2014

Posted by Trang Luong on 30-07-2014 - 15:23 in Thi HSG Quốc gia và Quốc tế

USA USAMO 2014

Ngày 1 : 29/4/2014

$\boxed 1$ Cho $a,b,c,d$ là các số thực thỏa mãn $b-d\geq 5$ và tất cả các số $x_1,x_2,x_3,x_4$  thuộc đa thức$P(x)=x^4+ax^3+bx^2+cx+d$ là số thực. Tìm giá trị nhỏ nhất có thể có của $(x_1^2+1)(x_2^2+1)(x_3^2+1)(x_4^2+1)$

$\boxed 2$ Gọi $\mathbb{Z}$ là tập hợp tất cả các số nguyên . Tìm tất cả các hàm số $f:\mathbb{Z}\rightarrow\mathbb{Z}$ thỏa mãn : $xf(2f(y)-x)+y^2f(2x-f(y))=\frac{f(x)^2}{x}+f(yf(y))$

với $x,y\in\mathbb{Z},x\neq 0$

$\boxed 3$ Chứng minh rằng : Có vô hạn tập hợp các điểm $\dots,\; P_{-3},\; P_{-2},\; P_{-1},\; P_0,\; P_1,\; P_2,\; P_3,\;\dots$ trên mặt phẳng thỏa mãn các điều kiện sau : Bất kỳ 3 số nguyên $a,b,c$ phân biệt và 3 điểm $P_a,P_b,P_c$ thẳng hàng khi và chỉ khi $a+b+c=2014$

Ngày 2 : 30/4/2014

$\boxed 4$ Gọi $k$ là số nguyên dương. Có 2 người chơi $A$ và $B$ chơi trên 1 hệ thống gồm vô số ô lưới hình lục giác đều. Ban đầu tất cả các ô lưới đều trống, sau đó các người chơi thay nhau di chuyển các ô. $A$ là người di chuyển đầu tiên. Trong lượt di chuyển của $A$, anh ấy chọn 2 ô hình lục giác đều trống gần kề nhau và  đưa đi một truy câp (counter) (*) trong cả hai . Trong lượt của $B$ anh ấy chọn bất kỳ 1 truy cập (counter) trong bảng và loại bỏ. Nếu bất cứ lúc nào có $k$ ô liên tiếp trong 1 đường chứa 1 truy câp, và $A$ thắng.Tìm giá trị nhỏ nhất của $k$ mà $A$ không thể giành chiến thắng trong một số hữu hạn di chuyển, hoặc chứng minh rằng không có giá trị tối thiểu như vậy tồn tại.

$\boxed 5$ Cho $\Delta ABC$ với trực tâm $H$ và $P$ là giao điểm thứ hai của đường tròn ngoại tiếp $\Delta AHC$ với đường phân giác trong của $\widehat{BAC}$. Gọi $X$ là tâm đường tròn ngoại tiếp $\Delta APB$ và $Y$ là trực tâm $\Delta APC$. Chứng minh rằng : độ dài đoạn $XY$ bằng độ dài bán kính đường tròn ngoại tiếp $\Delta ABC$

$\boxed 6$ Chứng minh rằng tồn tại một hằng số $c>0$ thỏa mãn : Nếu $a,b,n$ là các số nguyên dương sao cho $\gcd(a+i, b+j)>1$  với mọi $i, j\in\{0, 1,\ldots n\}$ thì $\min\{a, b\}>c^n\cdot n^{\frac{n}{2}}.$

 

P/s: (*) counter : mk dịch chưa sát nghĩa lắm, m,n cố gắng diễn đạt theo ý riêng của mỗi người nhé




#506285 Tuyển tập đề thi vào các trường $\boxed{\textrm{THPT Chuyên}}...

Posted by Trang Luong on 13-06-2014 - 15:00 in Tài liệu - Đề thi

Tuyển tập đề thi vào các trường THPT chuyên trên toàn quốc năm 2014 - 2015

 

 

P/s: Topic sẽ luôn được cập nhập mới. Mong các ĐHV THCS cập nhập thêm




#465812 Tuyển tập các bài hình thi vào chuyên THPT

Posted by Trang Luong on 21-11-2013 - 21:56 in Hình học

Cho hình vuông $ABCD$. Tìm tập hợp các điểm $M$ nằm trong tam giác (không năm trên cạnh hình vuông) sao cho $\widehat{BAM}+\widehat{BMC}+\widehat{MCD}+\widehat{MDA}=180^{\circ}$




#430577 Tuyển người làm MHS Shortlist, MSS Shortlist, MO Shortlist năm 2013

Posted by Trang Luong on 25-06-2013 - 20:16 in Thi giải toán Marathon cấp THPT 2013

Nhiệm vụ của MSS Shorlist là gì vậy ạ




#498117 Trận 9 - Bất đẳng thức

Posted by Trang Luong on 09-05-2014 - 21:25 in Thi giải toán Marathon cấp THCS 2014

Cho $x, y, z$ là các số dương thỏa mãn điều kiện $xyz = 1$. Tìm giá trị nhỏ nhất của biểu thức:  

$$E=\frac{1}{x^3(y+z)}+\frac{1}{y^3(z+x)}+\frac{1}{z^3(x+y)}.$$

Toán thủ ra đề: angleofdarkness

Đặt $x=\frac{1}{a},y=\frac{1}{b},z=\frac{1}{c}\Rightarrow abc=1$ vì $xyz=1$

Điều kiện : $a,b,c>0$

Ta có : $E=\frac{1}{x^3(y+z)}+\frac{1}{y^3(z+x)}+\frac{1}{z^3(x+y)}=\frac{1}{\frac{1}{a^3}\left ( \frac{1}{b}+\frac{1}{c} \right )}+\frac{1}{\frac{1}{c^3}\left ( \frac{1}{a}+\frac{1}{b} \right )}+\frac{1}{\frac{1}{b^3}\left ( \frac{1}{a}+\frac{1}{c} \right )}=\frac{a^{3}bc}{b+c}+\frac{b^{3}ac}{a+c}+\frac{c^{3}ab}{a+b}=\frac{a^2}{b+c}+\frac{b^2}{a+c}+\frac{c^2}{a+b}\geq \frac{(a+b+c)^{2}}{a+b+c}=a+b+c$

Theo BĐT Schwarz

mà $a+b+c\geq 3\sqrt[3]{abc}=3$ (Theo BĐT AM-GM)

Vậy $min E=3\Leftrightarrow \left\{\begin{matrix} a=b=c\\ abc=1 \end{matrix}\right.\Rightarrow a=b=c=1\Rightarrow x=y=z=1$

 

d = 9.5

S = 45




#498430 Trận 9 - Bất đẳng thức

Posted by Trang Luong on 11-05-2014 - 19:44 in Thi giải toán Marathon cấp THCS 2014

Mở rộng 1: Cho $xyz=1;x,y,z>0$

Tìm giá trị nhỏ nhất của : $P=\frac{1}{x^{3k}\left ( y^k+z^k \right )}+\frac{1}{y^{3k}\left ( x^k+z^k \right )}+\frac{1}{z^{3k}\left ( y^k+x^k \right )}$ với $k$ là số tự nhiên lớn hơn hoặc bằng 1

Bài giải :

Vì $x,y,z>0$, đặt : $x^k=\frac{1}{a},y^k=\frac{1}{b},z^k=\frac{1}{c}\Rightarrow x^{3k}=\frac{1}{a^3},y^{3k}=\frac{1}{b^3},z^{3k}=\frac{1}{c^3}$ với $a,b,c>0$

mà $xyz=1\Rightarrow abc=1$

Thay vào BĐT ,ta có BĐT mới là

$P=\frac{a^3bc}{b+c}+\frac{b^3ac}{a+c}+\frac{c^3ab}{a+b}=\frac{a^2}{b+c}+\frac{b^2}{a+c}+\frac{c^2}{a+b}\geq \frac{(a+b+c)^2}{a+b+c}=a+b+c\geq 3\sqrt[3]{abc}=3$

Áp dụng BĐT Scharw và BĐT AM-GM

Vậy $minP=3$ khi $a=b=c=1\Leftrightarrow x=y=z=1$




#492391 Trận 7 - PT, HPT đại số

Posted by Trang Luong on 12-04-2014 - 11:58 in Thi giải toán Marathon cấp THCS 2014

Mở rộng 2:

Cho $m,n>0;p\geq q\geq 0$. Giải phương trình : 

$\left ( mn+1 \right )\sqrt{p^x-q^x}=n\left ( p-q \right )+m\left ( p^{x-1}+p^{x-2}q+p^{x-3}q^2+...+q^{x-2}p+q^{x-1} \right )$ Với $x\geq 1$

Bài giải : Ta thấy : $p^x-q^x=\left ( p-q \right )\left ( p^{x-1}+p^{x-2}q+p^{x-3}q^2+...+q^{x-2}p+q^{x-1} \right )$

Đặt $\left ( p-q \right )=u,\left ( p^{x-1}+p^{x-2}q+p^{x-3}q^2+...+q^{x-2}p+q^{x-1} \right )=v$

Thao vào PT ban đầu $\Rightarrow nu^2+mv^2-(mn+1)uv=0\Leftrightarrow \left ( nu-v \right )\left ( u-mv \right )=0\Rightarrow \begin{bmatrix} nu-v=0\\ u-mv=0 \end{bmatrix}$

Ta xét các trường hợp : 




#492221 Trận 7 - PT, HPT đại số

Posted by Trang Luong on 11-04-2014 - 20:13 in Thi giải toán Marathon cấp THCS 2014

Giải phương trình: $2x^{2}+5x-1=7\sqrt{x^{3}-1}$

Đề thi của l4lzTeoz

Điều kiện $\left\{\begin{matrix} 2x^2+5x-1\geq 0\\ x^3-1\geq 0 \end{matrix}\right.\Leftrightarrow \left\{\begin{matrix} \begin{bmatrix} x\geq \frac{-5+\sqrt{33}}{4}\\ x\leq \frac{-5-\sqrt{33}}{4} \end{bmatrix}\\ x\geq 1 \end{matrix}\right.\Rightarrow x\geq 1$

Ta có :

$2x^{2}+5x-1=7\sqrt{x^{3}-1}\Leftrightarrow 2x^2+5x-1=7\sqrt{(x-1)(x^2+x+1)} (*)$

Đặt $\sqrt{x-1}=a,\sqrt{x^2+x+1}=b$ $\Rightarrow 2x^2+5x-1=2x^2+2x+2+3x-3=2b^2+3a^2$

Thay vào phương trình (*) 

Ta có$2b^2+3a^2-7ab=0\Leftrightarrow \left ( 2b-a \right )\left ( b-3a \right )=0\Rightarrow \begin{bmatrix} 2b-a=0\\ b-3a=0 \end{bmatrix}$

Với $2b-a=0\Rightarrow 2b=a\Rightarrow 4b^2=a^2\Leftrightarrow 4\left ( x^2+x+1 \right )=x-1\Rightarrow 4x^2+3x+5=0(VN)$ vì $\Delta =3^2-4.5.4=-71< 0$

Với $b-3a=0\Rightarrow b=3a\Rightarrow b^2=9a^2\Leftrightarrow x^2+x+1=9(x-1)\Rightarrow x^2-8x+10=0\Leftrightarrow \left ( x-4-\sqrt{6} \right )\left ( x-4+\sqrt{6} \right )=0\Rightarrow \begin{bmatrix} x=4+\sqrt{6}\\ x=4-\sqrt{6} \end{bmatrix}$ thỏa mãn điều kiện

Vậy $S=\left ( 4-\sqrt{6};4+\sqrt{6} \right )$

 

   d =10

  $d_{mr}=10$

  S =17+10.3+10=57




#492257 Trận 7 - PT, HPT đại số

Posted by Trang Luong on 11-04-2014 - 20:58 in Thi giải toán Marathon cấp THCS 2014

Giải phương trình: $2x^{2}+5x-1=7\sqrt{x^{3}-1}$

Đề thi của l4lzTeoz

Mở rộng 1:

Giải hệ phương trình : $nx^2+(m+n)x+(n-m)=\left ( mn+1 \right )\sqrt{x^3-1}$ với $m,n> 0$

Điều kiện $x\geq 1$

Ta có :$nx^2+(m+n)x+(n-m)=\left ( mn+1 \right )\sqrt{x^3-1}\Leftrightarrow n\left ( x^2+x+1 \right )+m\left ( x-1 \right )=\left ( mn+1 \right )\sqrt{\left ( x-1 \right )\left ( x^2+x+1 \right )} (*)$

Đặt $\sqrt{x-1}=a,\sqrt{x^2+x+1}=b\left ( a\geq 0,b> 0 \right )$

Thay vào PT  $(*)\Rightarrow ma^2+nb^2-(mn+1)ab=0\Leftrightarrow (ma-b)(a-nb)=0\Rightarrow \begin{bmatrix} ma-b=0\\ a-nb=0 \end{bmatrix}$

  • Với $ma-b=0$
  • Với $a-nb=0$

Tùy thuộc vào giá trị của $m,n$ thì ta tìm được nghiệm của $x$ 

 

  Mở rộng đúng




#485808 Trận 4 - Bất đẳng thức

Posted by Trang Luong on 04-03-2014 - 17:31 in Thi giải toán Marathon cấp THCS 2014

Cho $x,y\in \mathbb{R}$ thỏa mãn $(x+y)^{3}+4xy\geq 2$(*) . Tìm giá trị nhỏ nhất của:

 

$$ P=3(x^{4}+y^{4}+x^{2}y^{2})-2(x^{2}+y^{2})+1$$

Đề của 

nk0kckungtjnh

Bài làm:

P=3x4+3y4+3x2y2-2x2-2y2+$\frac{1}{3}$.3

P=3(x4-$\frac{2}{3}$x2+$\frac{1}{9}$)+3(y4-$\frac{2}{3}$y2+$\frac{1}{9}$)+3x2y2+$\frac{1}{3}$

P=3(x2-$\frac{1}{3}$)2+3(y2-$\frac{1}{3}$))2+3x2y2+$\frac{1}{3}$

Nhận thấy:3(x2-$\frac{1}{3}$)2+3(y2-$\frac{1}{3}$))2$\geq 0$

                 3x2y2$\geq 0$

Vậy nên để P có min thì cặp số x,y phải thoả mãn 1 trong hai điều kiện sau:3(x2-$\frac{1}{3}$)2+3(y2-$\frac{1}{3}$))2=0

                                                                                                                       3x2y2=0

 

*Trường hợp 1:3x2y2=0

Theo bài toán đầu đề thì ta nhận thấy x,y không thể cùng bằng 0

Mà theo trường hợp thì x=0 hoặc y=0

Giả sử x=0 và y=m,thay x=0 vào bài toán đầu bài(*) ta có được y$\geq$$\sqrt[3]{2}$

Thay x=0,y$\geq$$\sqrt[3]{2}$ vào P ta có=3(0-$\frac{1}{3}$)2+3(y2-$\frac{1}{3}$)2+0+$\frac{1}{3}$$\geq$5,4(1)

*Trường hợp 2 :3(x2-$\frac{1}{3}$)2+3(y2-$\frac{1}{3}$))2=0

=>x=$\sqrt{\frac{1}{3}}$ hoặc x=$-\sqrt{\frac{1}{3}}$và y=$-\sqrt{\frac{1}{3}}$ hoặc y=$\sqrt{\frac{1}{3}}$

vậy nên ta sẽ có được 4 cặp số thoả mãn trường hợp

Thay lần lượt 4 cặp số vừa tìm được vào bài toán đầu bài(*) thì ta chỉ nhận được 1 cặp số x,y thoả mãn là:

(x,y)=($\sqrt{\frac{1}{3}}$;$\sqrt{\frac{1}{3}}$)

Thay cặp x,y ở trên vào P ta có P$\geq \frac{2}{3}$(2)

Từ (1),(2) suy ra P$\geq \frac{2}{3}$ cho nên Min của P là $\frac{2}{3}$. dấu bằng xảy ra khi x=y=$\sqrt{\frac{1}{3}}$

Phần này sai. Vì bạn chọn nhầm điểm rơi.  $\left ( x-\frac{1}{3} \right )^{2}+\left ( y-\frac{1}{3} \right )^2+x^2y^2$ nhỏ nhất khj ba số bằng nhau theo cách của bạn




#485731 Trận 4 - Bất đẳng thức

Posted by Trang Luong on 03-03-2014 - 23:09 in Thi giải toán Marathon cấp THCS 2014

MSS-01: Nguyễn Đức Thuận

Giải:

Từ BĐT AM-GM, ta có: $xy\leq \frac{(x+y)^2}{4}$ đặt $m=x+y$

$\Rightarrow m^3+\frac{km^2}{4}\geq 1+\frac{k}{4}$$\Leftrightarrow (m-1)\left [ m^2-\left ( 1+\frac{k}{4}\right ) m+1+\frac{k}{4} \right ]$

Mà $m^2-\left ( 1+\frac{k}{4}\right ) m+1+\frac{k}{4} =\left [ m-\frac{1}{2}\left ( 1+\frac{k}{4} \right ) \right ]^2+1+\frac{k}{4}-\frac{1}{4}\left ( 1+\frac{k}{4} \right )^2>0$  

(Do $-4<k<12\Rightarrow 1+\frac{k}{4}<4\Rightarrow \frac{1}{4}\left ( 1+\frac{k}{4} \right )^2<1+\frac{k}{4}$)

Suy ra $m\geq 1$ $\Rightarrow t=x^2+y^2\geq \frac{m^2}{2}\geq \frac{1}{2}\Rightarrow t^2\geq \frac{1}{4}$

Áp dụng BĐT $x^2y^2\leq \frac{t^2}{4}$, biến đổi:

$P=a(t^2-x^2y^2)-bt+c\geq \frac{3a}{4}t^2-bt+c=b\left ( t-\frac{1}{2} \right )^2+\left ( \frac{3a}{4}-b \right )t^2+c-\frac{1}{4}$

$\geq \frac{1}{4}\left ( \frac{3a}{4}-b \right )+c-\frac{b}{4}$

 

Vậy $MinP= \frac{1}{4}\left ( \frac{3a}{4}-b \right )+c-\frac{b}{4}$   khi $x=y=1/2$

Nếu k<0 thì $(x+y)^{3}+kxy\geq (x+y)^{3}+k(x+y)^{2}$ trái dấu.




#485308 Trận 4 - Bất đẳng thức

Posted by Trang Luong on 01-03-2014 - 20:33 in Thi giải toán Marathon cấp THCS 2014

Theo giả thiết ta có : $(x+y)^3+4xy\geq 2\Rightarrow 2\leq (x+y)^3+(x+y)^2\Leftrightarrow (x+y)^3+(x+y)^2-2\geq 0\Leftrightarrow \left ( x+y-1 \right )\left [ \left ( x+y \right )^2+x+y+2 \right ]\geq 0\Rightarrow x+y\geq 1$

Ta có : $P=3\left ( x^4+y^4+x^2y^2 \right )-2\left ( x^2+y^2 \right )+1$

Áp dụng BĐT $AM-GM$ với các số thực $a,b$ ta có : $4ab\leq \left ( a+b \right )^2,a^2+b^2\geq 2\left | ab \right |\geq 2ab$

$\Rightarrow 2P=6\left ( x^4+y^4+x^2y^2 \right )-4\left ( x^2+y^2 \right )+2=2\left ( x^4+y^4-x^2y^2 \right )+4\left ( x^4+y^4+2x^2y^2 \right )-4\left ( x^2+y^2 \right )+2=2\left ( x^4+y^4-x^2y^2 \right )+\left [ 2\left ( x^2+y^2 \right )-1 \right ]+1$

$\geq \left ( x^4+y^4 \right )+\left ( x^4+y^4-2x^2y^2 \right )+\left [ \left ( x+y \right )^2 \right ]+1\geq x^4+y^4+1\geq 2.\left ( \frac{x+y}{4} \right )^4+1\geq \frac{9}{8}\Rightarrow P\geq \frac{9}{16}$

Dấu = xảy ra khi $x=y=\frac{1}{2}$

 

AM-GM chỉ xài khi có điều kiện không âm.

 

Điểm 9.